LSAT and Law School Admissions Forum

Get expert LSAT preparation and law school admissions advice from PowerScore Test Preparation.

 Echx73
  • Posts: 36
  • Joined: Nov 11, 2015
|
#24979
Hello,

I am wondering why A is the best answer? I crossed it off because since 85 was a wetter year there should be more movement of the beetles compared to a relatively dry year such as 89. Intuitively, this would suggest in my mind that the wetter the season the more you will see the beetle. Therefore, 85 would have more beetles than 89 which is not the case so I crossed A off. Also, can you tell me why B is incorrect? C, D and E are pretty self explanatory of why they are wrong.

Thanks!
Eric
 Adam Tyson
PowerScore Staff
  • PowerScore Staff
  • Posts: 5153
  • Joined: Apr 14, 2011
|
#25157
I think you may have answered your own question without realizing it, Eric! Your analysis of A is perfect - 85 was wetter, so the beetles were probably moving more and thus easier to spot. That explains why the observer saw so many more in that year than in 89, when they would have been moving less if at all. It looks like maybe you mis-read the stimulus, thinking the opposite occurred, that more were seen in 89 than in 85? Check it again and you will see the reverse is the case - in 85 it was 38 in 2 hours, in 89 it was only 10 in 9 hours.

So why does that make A the right answer? Focus on the conclusion - what is this author trying to prove? It's the beginning of the last sentence of the stimulus - "This difference probably does not reflect a drop in the population of these rare beetles over this period". In other words, he is saying that a drop in the population is likely NOT the reason for the lower number in 89. Per the stem, we want to strengthen that claim, and A does it by giving another explanation (an alternate cause, perhaps?) for the lower number - not that there were fewer beetles, but that there was less movement and thus they weren't as easily spotted.

B is incorrect because it does nothing to help that conclusion, that a drop in population is not the reason for the lower numbers seen in 89. B might actually weaken the argument by suggesting that in 89 the beetle habitat was reduced in size, thus reducing the population (the very thing we want to say did NOT happen). I'm more inclined to say that B simply has no impact - my reaction to it is "so what?" It tells us where their habitat is, but tells us nothing about whether this particular area is such a habitat and nothing about why, other than a population decrease, we saw fewer beetles in 89 than in 85.

Good work! Now just tie your analysis of the answer back to your goal, helping make the conclusion better, and you are home free.
 Joel T
  • Posts: 3
  • Joined: Apr 17, 2019
|
#64217
Hello,

I am having trouble understanding this one.

I chose E because I thought it eliminated an alternative possible explanation for why the beetle count was lower in 89 versus 85.

I know the stimulus explains that there was a drop because it was drier, but doesn't "no predators relies on delta green ground beetle for a major portion of its food supply" also eliminate that another alternative explanation?

Thanks for your help.
 Zach Foreman
PowerScore Staff
  • PowerScore Staff
  • Posts: 91
  • Joined: Apr 11, 2019
|
#64232
Although this is a strengthen question, we can use the Assumption Negation Technique to help us eliminate wrong answers. Why? Well, because an assumption does strengthen a conclusion and if we negate it and the conclusion is unaffected, then it isn't really relevant.
Let's start with the correct answer, A. What if the beetles were easily seen even when not moving? Wouldn't that totally undermine the argument? Then it wouldn't matter if it were wet or dry or if the beetles were active or inactive, the counter could just count them when stationary. So, we can see that assuming that they are difficult to see when stationary is an important fact and does strengthen the argument.
What about E? What if lots of predators relied on the beetles for food? Well, what difference would that make if we don't know whether there were more or less predators in 1989 vs 1985? The mere presence of predators wouldn't seem to make a difference. Therefore, it is irrelevant.
Now, if the answer had said "No new predator species invaded the territory since 1986" then that would eliminate an alternative cause.
Remember, you need a difference to explain a difference.

Difficulty in counting stationary beetles :arrow: Fewer beetles counted in 1989
The alternate cause being explicitly denied is population decline of beetles :arrow: Fewer beetles counted in 1989

What else was different between 1985 and 1989? We don't know. If another answer denied any other difference, it would be a correct answer. "There were no more predators counted in 1989 than 1985." or "1989 was not much colder/hotter than 1985." or "The habitat of the beetle did not shrank considerably in 1989." Any of these would eliminate an alternate cause. E suggests an alternate cause but because it doesn't say anything about a difference between 1989 and 1985 doesn't quite go far enough in eliminating an alternate cause.
 gmsanch3
  • Posts: 30
  • Joined: Oct 09, 2017
|
#79704
Hello, this thread helped me to finally understand why E is wrong, that said I’m now confused about how/when to use assumption negation technique as it was explained by Zach above that it could be used on this strength question. This is not the first time I’ve seen this used for a strength/justify question, but it was another company that used it for a non-assumption question so I ignored it because I try to strictly follow PowerScores question types and techniques for answering. It really helps me with assumption questions and I usually struggle with justify questions so: How do I know when to use assumption negation in a strength/justify question?
 Jeremy Press
PowerScore Staff
  • PowerScore Staff
  • Posts: 1000
  • Joined: Jun 12, 2017
|
#79749
Hi gm,

While I don't want to cause a big kerfuffle here among instructors, I'm going to recommend you stay away from the Assumption Negation Technique on Strengthen and Justify questions.

To understand Zach's reasoning here, you should know that there are occasions here and there where the same answer could be correct whether the question was an Assumption, Strengthen, or a Justify question. Take a simple example:

Premise: A passing score on the chemistry test is 60 out of 100 points.
Premise: I studied hard for the chemistry test and performed my best on the day of the test.
Conclusion: Therefore, I passed the chemistry test.

This same answer: "I got at least 60 out of 100 points on the chemistry test" would be correct whether the question was an Assumption (because it's necessary to get at least 60 points to pass), or a Strengthen/Justify (because getting at least 60 points out of 100 on the test would prove beyond a doubt that I passed the test, thus both strengthening and proving the conclusion in an absolute way).

Technically, the Assumption Negation Technique would "work" on such a question. But it's a bit of a gamble whether the question you're looking at truly has that kind of overlap.

Most Strengthen and Justify answers don't have that kind of overlap. And there are occasions where a Justify question, for example, has an Assumption answer. On such a question, if you negate the Assumption answer, it destroys the argument, and you'll think you should pick it. And if you negate the Justify answer, it doesn't destroy the argument, and you'll think you should eliminate it. That's then causing you to get the question wrong. A good example of that would be PT 75, June 2015, LR1 (Section 1), Question 23, a Justify question where answer choice B is an Assumption and answer choice C is a Justify.

So, in order to save confusion and to avoid potential mistakes, it's my personal recommendation that you should avoid the Assumption Negation Technique on Strengthen and Justify questions.

I hope this helps!
 gmsanch3
  • Posts: 30
  • Joined: Oct 09, 2017
|
#79950
Ok thank you... I appreciate that clarity. I will stick to Powerscore explanations on answering these question types. Thank you.
 LSAT2020
  • Posts: 31
  • Joined: Jun 24, 2020
|
#82516
I am still a little confused about why E is the correct answer. When reading this stimulus, I identified the conclusion to be:
"this difference probably doesn't reflect a drop in the population of these rare beetles over this period." I thought answer choice E would be enough to strengthen this conclusion since it blocks a possible loophole. As I was reading the explanation above, it emphasized that answer choice E did not take into account the fact that in 1985 there is a wet season and in 1989 there is a dry season. Why was it important that the correct answer choice address this? Thanks in advance!
User avatar
 KelseyWoods
PowerScore Staff
  • PowerScore Staff
  • Posts: 1079
  • Joined: Jun 26, 2013
|
#84021
Hi LSAT2020!

With Strengthen questions, focusing on the conclusion is important but you also want to think about the argument as a whole and specifically how the premises support that conclusion. Much of the time we are trying to strengthen that relationship between the premises and the conclusion. In this case, the author concludes that the difference in the number of beetles observed probably does not indicate a drop in the population. Why? Because 1985 was a wet year and 1989 was a dry year. This premise isn't fully connected to that conclusion leaving a gap in the argument. Why does whether the year was wet or dry matter? Usually if you have an argument with a gap like this, the way to strengthen the argument is to fill in that gap. We want to explain why the different in the wet/dry years would explain the difference in the number of beetles observed without it being an actual drop in population. Answer choice (A) fills in the gap by linking the premises more strongly to the conclusion.

Also, the conclusion is a little more nuanced than just that the beetles did not drop in population. It's that a drop in population is not the explanation for the change in the number of beetles observed and instead the difference in wet/dry years is the explanation. So we're not really just strengthening that there wasn't a drop in the beetle population, we're strengthening the alternative explanation the author has offered for the difference in the observations--that one year was wet and one was dry. Answer choice (A) strengthens that explanation.

Answer choice (E) just says that no predator relies on the beetle for a major portion of its food supply. But just because no one predator relies on the beetles as a major food source doesn't necessarily mean that the beetles don't have predators. And even if they don't have predators, that doesn't mean that their population still couldn't decrease for other reasons. And, again, we're really trying to strengthen the idea that the wet vs. dry year is the actual explanation for the difference in the observations. Answer choice (A) provides much stronger support for the argument and is, thus, the correct answer.

Hope this helps!

Best,
Kelsey

Get the most out of your LSAT Prep Plus subscription.

Analyze and track your performance with our Testing and Analytics Package.